3
$\begingroup$

enter image description here

Hi, How to convert $|100\rangle$ 3-qubit quantum state into $\frac{1}{\sqrt{2}}(|000\rangle + |111\rangle)$ state using only Hadamard and CNOT gates? Also, is output state an entangled one?

$\endgroup$

2 Answers 2

4
$\begingroup$

If the first qubit is in state $|1\rangle$, i.e. the input state $|100\rangle$ then resulting GHZ state is $\frac{1}{\sqrt{2}}(|000\rangle - |111\rangle)$, i.e. the phase is $\pi$. To have phase $0$, $Z$ has to be applied but this gate is not allowed. But you can use controlled $Z$ which is composed only with $H$ and $CNOT$. The circuit is this

enter image description here

A part before orange line produces the state $\frac{1}{\sqrt{2}}(|000\rangle - |111\rangle)$ from $|100\rangle$, a part after the line is controlled $Z$.

Here is a resulting state from IBM Q:

enter image description here

As you can see, the phase is correct.

$\endgroup$
2
$\begingroup$

enter image description here

The upper figure shows the quantum circuit to do so, the first $X$ gate is to let the first qubit be in the $|1\rangle$ state coherently.

The state $\frac{|000\rangle+|111\rangle}{\sqrt2}$ is the GHZ state, it is one of the most famous tri-qubit entangled state (another one is the W state), you may see Wikipedia for detail.

$\endgroup$
10
  • $\begingroup$ Hi Yitian, the circuit above produces |000>-|111>, but, I need |000>+|111> state. Phase is incorrect. I understand the confusion, considering you're viewing only probabilities. So, how to generate |000>+|111> ?? $\endgroup$
    – Sscr
    Nov 15, 2020 at 7:28
  • $\begingroup$ Note that $X$ gate is forbidden, only H and CNOT are allowed. $\endgroup$ Nov 15, 2020 at 7:39
  • 1
    $\begingroup$ Notice that if the X gate were allowed then it is very simple.... all you have to do is to bit flip the qubit to get it back to the state $|00\cdots 0\rangle$ and generate the GHZ states from there.. $\endgroup$
    – KAJ226
    Nov 15, 2020 at 7:53
  • 1
    $\begingroup$ Also if the Z gate is available then you don't have to do controlled Z either... you just need to apply the $Z$ gate to the top qubit to fix the phase. But without the Z gate, @MartinVesely is right! +1 $\endgroup$
    – KAJ226
    Nov 15, 2020 at 8:00
  • 1
    $\begingroup$ @MartinVesely You are absolutely correct. I just made my comments in a general setting... nothing more. :) $\endgroup$
    – KAJ226
    Nov 16, 2020 at 16:19

Your Answer

By clicking “Post Your Answer”, you agree to our terms of service and acknowledge you have read our privacy policy.

Not the answer you're looking for? Browse other questions tagged or ask your own question.